¿Por qué la helicidad de las partículas sin masa es invariante de Lorentz?

Por definición, la helicidad es la proyección del espín sobre el impulso 3.

h = j PAG
dónde PAG = ( PAG 1 , PAG 2 , PAG 3 ) es el operador de cantidad de movimiento y j = ( j 1 , j 2 , j 3 ) el operador angular.

Ahora, bajo una transformación de Lorentz, las partículas sin masa se transforman así:

tu ( Λ ) | pag , σ = mi i θ σ | Λ pag , σ .

Como podemos ver, el impulso está cambiando pero el giro no.

Supongamos que ese estado | pag , σ es un estado de helicidad σ tal que tenemos

h | pag , σ = j 3 PAG 3 | pag , σ = σ pag 3 | pag , σ

Pero para el estado tu ( Λ ) | pag , σ = mi i θ σ | Λ pag , σ , tendríamos

h | Λ pag , σ = σ pag 3 mi i θ σ | Λ pag , σ |
Así que para la conservación de la helicidad requeriríamos pag 3 = pag 3 lo cual no es siempre el caso.

Entonces, ¿por qué la gente dice que la helicidad es invariante de Lorentz?

WP .
La respuesta en el enlace que diste es buena, pero la definición de helicidad es diferente.

Respuestas (1)

Su fórmula para el operador de helicidad es incorrecta ; esto ya debería quedar claro a nivel de análisis dimensional. La fórmula correcta es (cf. Refs. 1 y 2)

h = j PAG | PAG |
dónde | PAG | denota la norma de PAG . Actuando en su estado con h no da factores de pag 3 , y así se resuelve la "paradoja".

Referencias.

  1. Schwartz - Teoría cuántica de campos y el modelo estándar §11.1.

  2. Ticciati - Teoría cuántica de campos para matemáticos §7.8.

@amiltonmoreira ¿Puedes intentarlo de nuevo, por favor? No tengo idea de lo que estás tratando de decir. (Primero tienes un operador, luego un estado, luego un producto tensorial de estados; ninguno de estos puede ser igual entre sí...)
lo siento problema con la computadora
Supongamos que tenemos j PAG | PAG | | pag , σ = σ | pag , σ . Supongamos ahora que nuestra transformación es una rotación en el X eje que j PAG | PAG | | Λ pag , σ = ( pag 2 j 2 pag 3 + pag 3 σ pag 3 ) | Λ pag , σ . Es ( pag 2 j 2 pag 3 + pag 3 σ pag 3 ) | Λ pag , σ igual a σ | Λ pag , σ ?
j PAG | PAG | es evidentemente un escalar bajo rotaciones... La elicidad no puede cambiar bajo rotaciones, esto también es cierto si la partícula es masiva.
¿Cómo puedo probar eso?
Simplemente aplique la representación unitaria de rotaciones y use el hecho de que j y PAG son 3-vectores bajo la acción de él mientras que | PAG | es un escalar....